Electricity

You might also like

Download as pdf or txt
Download as pdf or txt
You are on page 1of 78

ELECTRICITY

CHARGE, POTENTIAL DIFFERENCE AND WORK DONE


1. Define charge. When does a body said to be negatively charged and positively charged?
2. What is the unit of electric charge?
3. Define one coulomb charge.
4. What is meant by conductors and insulators? Give two examples of each.
5. Which of the following are conductors and which are insulators?
Sulphur, silver, copper, cotton, aluminium, air, nichrome, graphite, paper, porcelain,
mercury, mica, Bakelite, polythene, manganin.
6. What do you understand by the term “electric potential” or “potential at a point”? What is
the unit of electric potential?
7. What do you understand by the term “potential difference”?
8. What is meant by saying that the electric potential at a point is 1 𝑣𝑜𝑙𝑡?
9. What is meant by saying that the potential difference between two points is 1 𝑉?
10. What is the SI unit of potential difference?
11. State the relation between potential difference, work done and charge moved.
12. By what other name is the unit 𝒋𝒐𝒖𝒍𝒆/𝒄𝒐𝒖𝒍𝒐𝒎𝒃 called?
13. How much work is done when one coulomb charge moves against a potential difference of
1 𝑣𝑜𝑙𝑡?
14. Name a device which helps to maintain potential difference across a conductor (ex: a bulb).
15. Name a device which helps to measure potential difference across a conductor (ex: a bulb).
16. What is a voltmeter? How is a voltmeter connected in the circuit to measure the potential
difference between two points? Explain with the help of a diagram.
17. The atoms of copper contain electrons and the atoms of rubber also contain electrons.
Then why does copper conduct electricity but rubber does not conduct electricity?
18. Calculate the number of electrons constituting one coulomb of charge.
19. How much work is done in moving a charge of 2 𝐶 across two points having a potential
difference of 12 𝑉?
20. What is the potential difference between the terminals of a battery if 250 𝑗𝑜𝑢𝑙𝑒𝑠 of work is
required to transfer 20 𝑐𝑜𝑢𝑙𝑜𝑚𝑏𝑠 of charge from one terminal of battery to the other?
21. How much work is done in moving a charge of 2 𝑐𝑜𝑢𝑙𝑜𝑚𝑏𝑠 from a point at 118 𝑣𝑜𝑙𝑡𝑠 to a
point at 128 𝑣𝑜𝑙𝑡𝑠?
22. Calculate the work done in moving a charge of 4 coulombs from a point of 220 𝑣𝑜𝑙𝑡𝑠 to
another point at 230 𝑣𝑜𝑙𝑡𝑠.
23. How much energy is given to each 𝑐𝑜𝑢𝑙𝑜𝑚𝑏 of charge passing through a 6 𝑉 battery?
24. How much energy is transferred by a 12 𝑉 power supply to each coulomb of charge which
it moves around a circuit?
25. Three 2 𝑽 cells are connected in series and used as a battery in a circuit.
a) What is the p.d. at the terminals of the battery?
b) How many 𝒋𝒐𝒖𝒍𝒆𝒔 of electrical energy does 1 𝑪 gain on passing through
i) One cell? ii) All three cells?

ELECTRIC CURRENT AND CIRCUIT DIAGRAMS


26. What is the flow of charge called?
27. What is an electric current? What makes an electric current flow in a wire?
28. In which direction does conventional current flow around a circuit?
29. In which direction do electrons flow?
30. What actually travels through the wires when you switch on a light?
31. Which particles constitute the electric current in a metallic conductor?
32. Write down the formula which relates electric charge, time and electric current.
33. Define the unit of current?
34. By what name is the physical quantity 𝒄𝒐𝒖𝒍𝒐𝒎𝒃/𝒔𝒆𝒄𝒐𝒏𝒅 called?
35. One 𝑐𝑜𝑢𝑙𝑜𝑚𝑏 of charge flows through any cross-section of a conductor in 1 𝑠𝑒𝑐𝑜𝑛𝑑. What
is the current flowing through the conductor?
36. How many milliamperes are there in 1 𝑎𝑚𝑝𝑒𝑟𝑒?
37. How many microamperes are there in 1 𝑎𝑚𝑝𝑒𝑟𝑒?
38. Which instrument is used to measure electric current?
39. What is an ammeter? How is it connected in a circuit? Draw a diagram to illustrate your
answer.
40. Why should the resistance of an ammeter be very small and of a voltmeter be very large?
41. What does an electric circuit mean?
42. Draw the labelled diagram of an electric circuit comprising of a cell, a resistor, an ammeter,
a voltmeter and a closed switch.
43. An electric circuit consisting of a 0.5 𝑚 long nichrome wire 𝑋𝑌, an ammeter, a voltmeter,
four cells of 1.5 𝑉 each and a plug key was set up. Draw a diagram of this electric circuit to
study the relation between the potential difference maintained between the
points ′𝑋′ and ′𝑌′ and the electric current flowing through 𝑋𝑌.
44. A student made an electric circuit (shown below) to measure the current through two
lamps.

a) Are the lamps in series or parallel?


b) The student has made a mistake in this circuit. What is the mistake?
c) Draw a circuit diagram to show the correct way to connect the circuit. Use the proper
circuit symbols in your diagram.
45. A child has drawn the electrical circuit to study ohm’s law as shown in the figure. His
teacher told that the circuit diagram needs correction. Study the circuit diagram and
redraw it after making all corrections.

46. Draw a circuit diagram to show how 3 bulbs can be lit from a battery so that 2 bulbs are
controlled by the same switch while the third bulb has its own switch.
47. If 20 𝐶 of charge pass a point in a circuit in 1 𝑠𝑒𝑐𝑜𝑛𝑑, what current is flowing?
48. What is the current in a circuit if the charge passing each point is 20 𝐶 in 40 𝑠𝑒𝑐𝑜𝑛𝑑𝑠?
49. An electric bulb draws a current of 0.25 𝐴 for 20 𝑚𝑖𝑛𝑢𝑡𝑒𝑠. Calculate the amount of electric
charge that flows through the circuit.
50. A current of 4 𝐴 flows around a circuit for 10 𝑠𝑒𝑐𝑜𝑛𝑑𝑠. How much charge flows past a point
in a circuit in this time?
51. A radio set draws a current of 0.36 A for 15 minutes. Calculate the amount of electric
charge that flows through the circuit.
52. In 10 𝑠, a charge of 25 C leaves a battery, and 200 J of energy are delivered to an outside
circuit as a result. What is p.d. across the battery and what current flows from the battery?
53. A flash of lightning carries 10 C of charge which flows for 0.01 s. What is the current? If
the voltage is 10 MV, what is the energy?
54. How many electrons should pass through a conductor in 1 second to constitute 1 A
current?
55. How many electrons are flowing per second past a point in a circuit in which there is a
current of 5 amp?
56. The p.d. across a lamp is 12 V. How many joules of electrical energy are changed into
heat and light when:
a) a charge of 1 C passes through it?
b) a charge of 5 C passes through it?
c) a current of 2 A flows through it for 10 seconds?
57. If a potential difference of 10 V causes a current of 2 A to flow for 1 minute, how much
energy is transferred?
58. An electric heater is connected to the 230 V mains supply. A current of 8 A flows through
the heater.
a) How much charge flows around the circuit each second?
b) How much energy is transferred to the heater in 5 seconds?
O H M’ S L A W
59. Name the law which relates the current in a conductor to the potential difference across its
ends.
60. What is ohm’s law? Explain how it is used to define the unit of resistance.
61. How ohm’s law can be verified experimentally? Does it hold well under all conditions?
Comment.
62. Define the unit of resistance.
63. What is the ratio of potential difference and current known as?
64. What is meant by the resistance of a conductor? Write the relation between resistance,
potential difference and current.
65. Name the unit of electrical resistance and give its symbol.
66. Name the physical quantity whose unit is “ohm”.
67. The values of potential difference ‘V’ applied across a resistor and the corresponding values
of current 'I' flowing in the resistor are given below. Plot a graph between ‘V’ and ′I' and
calculate the resistance of the resistor.
Potential difference, V (in volts) 1.7 3.4 6.8 10.2 13.6
Current, I (in amperes) 0.5 1.0 2.0 3.0 4.0

68. The graph between V and 𝑰 for a conductor is a straight line passing through the origin.
a) Which law is illustrated by such a graph?
b) What should remain constant in a statement of this law?
69. Following graph was plotted between ‘V’ and '𝐼' values. What would be the values of
V
ratios when the potential difference is 0.8 V, 1.2 V and 1.6 V respectively?
𝐼
70. State the factors on which the strength of electric current flowing in a given conductor
depends.
71. Let the resistance of an electrical component remain constant while the potential
difference across the two ends of the component decreases to half of its former value.
What change will occur in the current through it?
72. Keeping the potential difference constant, the resistance of a circuit is halved. By how
much does the current change?
73. Keeping the potential difference constant, the resistance of a circuit is doubled. By how
much does the current change?
74. What is the general name of the substances having infinitely high electrical resistance?
75. Distinguish between good conductors, resistors and insulators, Name two good conductors,
two resistors and two insulators.
76. Classify the following into good conductors, resistors and insulators: Rubber, Mercury,
Nichrome, Polythene, Aluminium, Wood, Manganin, Bakelite, Iron, Paper, Thermocol,
Metal coin.
77. Why do electricians wear rubber hand gloves while working with electricity?
78. Name the material which is the best conductor of electricity.
79. Potential difference between two points of a wire carrying 2 ampere current is 0.1 volt.
Calculate the resistance between these points.
80. A simple electric circuit has a 24 V battery and a resistor of 60 ohms. What will be the
current in the circuit? The resistance of the connecting wires is negligible.
81. An electric iron draws a current of 3.4 A from the 220 V supply line. What current will this
electric iron draw when connected to 110 V supply line?
82. A potential difference of 20 V is applied across the ends of a resistance of 5 Ω. What
current will flow in the resistance?
83. A resistance of 20 ohms has a current of 2 amperes flowing in it. What potential difference
is there between its ends?
84. A current of 5 amperes flows through a wire whose ends are at a potential difference of 3
volts. Calculate the resistance of the wire.
85. When a 12 V battery is connected across an unknown resistor, there is a current of 2.5 mA
in the circuit. Calculate the value of the resistance of the resistor.
86. What p.d. is needed to send a current of 6 A through an electrical appliance having a
resistance of 40 Ω?
87. The potential difference between the terminals of an electric iron is 240 V and the current
is 5 A. What is the resistance of the electric iron?
88. An electric room heater draws a current of 2.4 A from the 120 V supply line. What current
will this room heater draw when connected to 240 supply line?
89. A p.d. of 10 V is needed to make a current of 0.02 A flow through a wire. What p.d. is
needed to make a current of 250 mA flow through the same wire?
90. A current of 200 mA flows through a 4 𝒌𝛀 resistor. What is the p.d. across the resistor?

RESISTANCE AND RESISTIVITY


91. Define resistance. On what factors does the resistance of a conductor depend?
92. How does the resistance of a conductor depend on the –
a) Length of the conductor?
b) Area of cross-section of the conductor?
c) Temperature of the conductor?
d) Nature of material of the conductor?
93. Give an example to show how the resistance depends on the nature of material of the
conductor.
94. What would be the effect on the resistance of a metal wire of
a) Increasing its length?
b) Increasing its diameter?
c) Increasing its temperature?
95. What happens to the resistance as the conductor is made thinner?
96. What happens to the resistance as the conductor is made thicker?
97. Which has less electrical resistance: a thin wire or a thick wire (of the same length and
same material)?
98. Will current flow more easily through a thick wire or a thin wire of the same material when
connected to the same source? Why?
99. If the length of a wire is doubled by taking more of wire, what happens to its resistance?
100. Describe an experiment to study the factors on which the resistance of a conducting wire
depends.
101. Define resistivity. Write an expression for the resistivity of a substance. Give the meaning
of each symbols which occurs in it.
102. Write down an expression for the resistance of a metallic wire in terms of resistivity.
103. State the S.I. unit of resistivity.
104. Name two factors on which the resistivity of a substance depends and two factors on which
it does not depends.
105. Distinguish between resistance and resistivity.
106. How does the resistance of a pure metal change if its temperature decreases?
107. How does the resistance of a wire change when
a) Its length is tripled?
b) Its diameter is tripled?
c) Its material is changed to one whose resistivity is three times?
108. The electrical resistivities of 3 materials P, Q and R are given below:

P 2.3 × 𝟏𝟎𝟑 𝛀𝐦

Q 2.63 × 𝟏𝟎−𝟖 𝛀𝐦
R 1.0 × 𝟏𝟎𝟏𝟓 𝛀𝐦
Which material will you use for making –
a) Electric wires?
b) Handle for soldering iron?
c) Solar cells?
109. The electrical resistivities of 4 materials A, B, C and D are given below:

A 110 × 𝟏𝟎−𝟖 𝛀𝐦

B 1.0 × 𝟏𝟎𝟏𝟎 𝛀𝐦
C 10.0 × 𝟏𝟎−𝟖 𝛀𝐦
D 2.3 × 𝟏𝟎𝟑 𝛀𝐦
Which material is –
a) Good conductor?
b) Resistor?
c) Insulator?
d) Semi-conductor?
110. The electrical resistivities of 5 substances A, B, C, D and E are given below:

A 5.20 × 𝟏𝟎−𝟖 𝛀𝐦

B 110 × 𝟏𝟎−𝟖 𝛀𝐦
C 2.60 × 𝟏𝟎−𝟖 𝛀𝐦
D 10.0 × 𝟏𝟎−𝟖 𝛀𝐦
E 1.70 × 𝟏𝟎−𝟖 𝛀𝐦
a) Which substance is the best conductor of electricity? Why?
b) Which one is the better conductor: A or C? Why?
c) Which substance would you advise to be used for making heating elements of
electric irons? Why?
d) Which two substances should be used for making electric wires? Why?
111. Give two examples of substances which are good conductors of electricity. Why do you
think that they are good conductors of electricity?
112. Which among iron and mercury is a better conductor of electricity.
113. Why are copper and aluminium wires usually used for electricity transmission?
114. Name the material which is used for making the heating element of an electric iron.
115. What is Nichrome? State its one use.
116. How does the presence of impurities in a metal affect its resistance?
117. Give two reasons why nichrome alloy is used for making the heating elements of
electrical appliances?
118. Why are coils of electric irons and electric toasters made of an alloy rather than a pure
metal?
119. A copper wire of length 2 𝑚 and area of cross-section 1.7 × 10−6 𝑚2 has a resistance
of 2 × 10−2 ohms. Calculate the resistivity of copper.
120. A copper wire has a diameter of 0.5 𝑚𝑚 and resistivity of 1.6 × 10−8 Ωm.
a) What will be the length of this wire to make its resistance 10 Ω?
b) How much does the resistance change if the diameter is doubled?
121. A 6 Ω resistance wire is doubled up by folding. Calculate the new resistance of the wire.
122. A wire is 1.0 𝑚 long, 0.2 𝑚𝑚 in diameter and have a resistance of 10 Ω. Calculate the
resistivity of its material.
123. What will be the resistance of a metal wire of length 2 𝑚 and area of cross-section
1.55 × 10−6 𝑚2 , if the resistivity of the metal be 2.8 × 10−8 Ωm?
124. Calculate the resistance of a copper wire 1.0 𝑘𝑚 long and 0.50 𝑚𝑚 diameter if the
resistivity of copper is 1.7 × 10−8 Ωm.
125. Calculate the resistance of an aluminium cable of length 10 𝑘𝑚 and diameter 2.0 𝑚𝑚 if the
resistivity of aluminium is 2.7 × 10−8 Ωm.
126. Calculate the area of cross-section of a wire if its length is 1.0 𝑚, its resistance is 23 Ω and
the resistivity of the material of the wire is 1.84 × 10−6 Ωm.
127. The resistance of a metal wire of length 1 𝒎 is 26 𝛀 at 20° C. If the diameter of the wire is
0.3 𝒎𝒎, what will be the resistivity of the metal at that temperature?
128. A piece of wire of resistance 20 𝛀 is drawn out so that its length is increased to twice its
original length. Calculate the resistance of the wire in the new situation.

RESISTORS CONNECTED IN SERIES AND PARALLEL


129. Give the law of combination of resistances in series.
130. Give the law of combination of resistances in parallel.
131. With the help of a circuit diagram, deduce the equivalent resistance of two resistances
connected in series.
132. With the help of a diagram, derive the formula for the resultant resistance of three
resistances connected in series.
133. With the help of a circuit diagram, obtain the relation for the equivalent resistance of two
resistances connected in parallel.
134. Explain with the help of a labelled circuit diagram, how you will find the resistance of a
combination of three resistors of resistances 𝑅1 , 𝑅2 and 𝑅3 joined in parallel.
135. How will you infer with the help of an experiment that the same current flows through
every part of the circuit containing three resistances in series connected to a battery?
136. How will you conclude that the same potential difference exists across three resistors
connected in a parallel arrangement to a battery?
137. If 5 resistances each of value 0.2 ohm, are connected in series, what will be the resultant
resistance?
138. If 3 resistances of 3 ohm each are connected in parallel, what will be their total
resistance?
139. Two resistances X and Y are connected turn by turn in parallel, and in series. In which
case the resultant resistance will be less than either of the individual resistances?
140. Judge the equivalent resistance when the resistors of 1 𝛀, 𝟏𝟎𝟑 𝛀 and 𝟏𝟎𝟔 𝛀 are
connected in parallel.
141. Two resistors with resistances 5 𝛀 and 10 𝛀 respectively are to be connected to a battery
of 6 V. How will you connect the resistances so as to obtain:
a) Minimum current flowing? b) Maximum current flowing?
142. What possible values of resultant resistance one can get by combining two resistances,
one of value 2 ohm and the other 6 ohm?
143. How should the two resistances of 2 ohms each be connected so as to produce an
equivalent resistance of 1 ohm?
144. Show how you would connect two 4 ohm resistors to produce a combined resistance of:
a) 2 ohms. b) 8 ohms.
145. How can 3 resistors of resistances 2 𝛀, 3 𝛀 and 6 𝛀 be connected to get a total resistance
of:
a) 4 𝛀. b) 1 𝛀.
146. Show how you would connect 3 resistors, each of resistance 6 𝛀, so that the combination
has a resistance of:
a) 9 𝛀. b) 4 𝛀.
147. How will you connect 3 resistors of 2 𝛀, 3 𝛀 and 5 𝛀 respectively so as to obtain a
resultant resistance of 2.5 𝛀?
148. You are given 3 resistances of 1, 2 and 3 ohms. How would you connect them to get total
resistance:
a) 6 𝛀. 𝟔 c) 1.5 𝛀.
b) 𝟏𝟏
𝛀.

149. You are supplied with a number of 100 𝛀 resistors. How could you combine some of
these resistors to make a 250 𝛀 resistor?
150. By the combination of four coils of resistances 4 𝛀, 8 𝛀, 12 𝛀 and 24 𝛀, find:
a) Highest total resistance possible. b) Lowest total resistance possible.
151. You are given one hundred 1 𝛀 resistors. What is the smallest and largest resistance you
can make in a circuit using these?
152. A resistor of 8 ohms is connected in parallel with another resistor X. The resultant
resistance of the combination is 4.8 ohms. What is the value of the resistor X?
153. Two resistances when connected in parallel give resultant value of 2 ohms; when
connected in series the value becomes 9 ohms. Calculate the value of each resistance.
154. How many 176 𝛀 resistors in parallel are required to carry 5 A on a 220 V line?
155. A wire that has resistance R is cut into two equal pieces. The two parts are joined in
parallel. What is the resistance of the combination?
156. A wire of resistance R is cut into five equal pieces. These five pieces of wire are then
connected in parallel. If the resultant resistance of this combination be R’, then what is
the ratio of R : R’?
157. Calculate the combined resistance in each case:

158. A hot plate of an electric oven connected to a 220 V line has two resistance coils A and B,
each of 24 Ω resistance, which may be used separately, in series or in parallel. What are the
currents in the three cases?
159. A battery of 9 V is connected in series with resistors of 0.2 Ω, 0.3 Ω, 0.4 Ω, 0.5 Ω and 12 Ω.
How much current would flow through the 12 Ω resistor?
160. The figure given below shows an electric circuit in which current flows from a 6 V battery
through two resistors.

a) Are the resistors connected in series with each other or in parallel?


b) For each resistor, state the p.d. across it.
c) The current flowing from the battery is shared between the two resistors. Which
resistor will have bigger share of the current?
d) Calculate the effective resistance of the two resistors.
e) Calculate the current that flows from the battery.
161. Find the current in each resistor in the circuit shown below:

162. Draw a circuit diagram of an electric circuit containing a cell, a key, an ammeter, a resistor
of 2 Ω in series with a combination of two resistors of 4 Ω each in parallel and a voltmeter
across the parallel combination. Will the potential difference across the 2 Ω resistor be the
same as that across the parallel combination of 4 Ω resistors? Give reason.
163. A current of 1 A flows in a series circuit containing an electric lamp and a conductor of
5 𝛀 when connected to a 10 V battery. Calculate the resistance of the electric lamp. Now,
if a resistance of 10 𝛀 is connected in parallel with this series combination, what change
in current flowing through 5 𝛀 conductor and potential difference across the lamp will
take place?
164. An electric lamp of resistance 100 𝛀, a toaster of resistance 50 𝛀, and a water filter of
resistance 500 𝛀 are connected in parallel to a 220 V source. What is the resistance of an
electric iron connected to the same source that takes as much current as all three
appliances, and what is the current through it?
165. A 4 Ω coil and a 2 Ω coil are connected in parallel. What is their combined resistance? A
total current of 3 A passes through the coils. What current passes through the 2 Ω coil?
166. An electric bulb of resistance 20 Ω and a resistance wire of 4 Ω are connected in series with
a 6 V battery. Draw the circuit diagram and calculate:
a) Total resistance of the circuit.
b) Current through the circuit.
c) Potential difference across the electric bulb.
d) Potential difference across the resistance wire.
167. A 5 V battery is connected to two 20 Ω resistors which are joined together in series.
a) Draw a circuit diagram to represent this. Add an arrow to indicate the direction of
conventional current flow in the circuit.
b) What is the effective resistance of the two resistors?
c) Calculate the current that flows from the battery.
d) What is the p.d. across each resistor?
168. A p.d. of 6 V applied to two resistors of 3 Ω and 6 Ω connected in parallel. Calculate:
a) the combined resistance
b) the current flowing in the main circuit
c) the current flowing in the 3 Ω resistor
169. A p.d. of 4 V is applied to two resistors of 6 Ω and 2 Ω connected in series. Calculate:
a) the combined resistance
b) the current flowing
c) the p.d. across the 6 Ω resistor
170. An electric heater which is connected to a 220 V supply line has two resistance coils A and
B of 24 Ω resistance each. These coils can be used separately (one at a time), in series or in
parallel. Calculate the current drawn when:
a) Only one coil A is used.
b) Coils A and B are used in series.
c) Coils A and B are used in parallel.
171. Three resistors are connected as shown in the diagram. Through the resistor 5 ohm, a
current of 1 ampere is flowing.

a) What is the current through the other two resistors?


b) What is the p.d. across AB and across AC?
c) What is the total resistance?
172. For the circuit shown in the diagram below, what is the value of:
a) Current through 6 Ω resistor?
b) Potential difference across 12 Ω resistor?
173. In the circuit diagram given below, find:

a) Total resistance of the circuit.


b) Total current flowing in the circuit.
c) The potential difference across 𝑅1 .
174. In the circuit diagram given below, three resistors 𝑅1 , 𝑅2 and 𝑅3 of 5 Ω, 10 Ω and
30 Ω respectively are connected as shown. Calculate:

a) Current through each resistor.


b) Total current in the circuit.
c) Total resistance in the circuit.
175. In the circuit diagram given below, calculate:
a) Total resistance in the circuit.
b) Total current flowing in the circuit.
176. In the circuit shown below, the voltmeter reads 10 V.

a) What is the combined resistance?


b) What current flows?
c) What is the p.d. across 2 Ω resistor?
d) What is the p.d. across 3 Ω resistor?
177. In the circuit given below:

a) What is the combined resistance?


b) What is the p.d. across the combined resistance?
c) What is the p.d. across the 3 Ω resistor?
d) What is the current in the 3 Ω resistor?
e) What is the current in the 6 Ω resistor?
178. In the circuit diagram given below, the current flowing across 5 ohm resistor is 1 amp. Find
the current flowing through the other two resistors.
179. In the circuit diagram shown below:

a) What is the current through the 5 ohm resistance?


b) What is the current through R?
c) What is the value of R?
d) What is the value of V?
180. For the circuit shown in the diagram given below, calculate:

a) The value of current through each resistor.


b) The total current in the circuit.
c) The total effective resistance of the circuit.
181. In the circuit diagram, shown below, find:
a) Total resistance.
b) Current shown by the ammeter A.
182. Four resistances of 16 ohms each are connected in parallel. Four such combinations are
connected in series. What is the total resistance?
183. In the diagram shown below, the cell and the ammeter both have negligible resistance.
The resistors are identical. With the switch K open, the ammeter reads 0.6 A. What will
be the ammeter reading when the switch is closed?

184. If the lamps are both the same in the figure given below and if 𝑨𝟏 reads 0.50 A, what
do 𝑨𝟐 , 𝑨𝟑 , 𝑨𝟒 and 𝑨𝟓 read?
DOMESTIC ELECTRIC CIRCUITS:SERIES OR PARALLEL
185. What are the advantages of connecting electrical devices in parallel with the battery
instead of connecting them in series?
Or
What are the disadvantages of connecting electrical devices in series with the battery?
186. Give four reasons why is series arrangement not used for connecting domestic electrical
appliances in a circuit?
187. Give four reasons why different electrical appliances in a domestic circuit are connected in
parallel?
188. What happens to the other bulbs in a series circuit if one bulb blows off?
189. What happens to the other bulbs in a parallel circuit if one bulb blows off?
190. Christmas tree lamps are usually wired in series. What happens if one lamp breaks?
191. An electrician has wired a house in such a way that if a lamp gets fused in one room of the
house, all the lamps in other rooms of the house stop working. What is the defect in the
wiring?
192. Are the lights in your house wired in series?
193. If you were going to connect two light bulbs to one battery, would you use a series or a
parallel arrangement? Why?
194. Which type of circuit, series or parallel, is preferred while connecting a large number of
bulbs for –
a) Decorating a hotel building from outside?
b) Lighting inside the rooms of a hotel?
195. Which arrangement takes more current from the battery: series or parallel?
196. How do you think the brightness of two lamps arranged in parallel compares with the
brightness of two lamps arranged in series (both arrangements having one cell)?
197. Consider the circuits given below:

a) In which circuit are the lamps dimmest?


b) In which circuit or circuits are the lamps of equal brightness to the lamps in circuit (i)?
c) Which circuit gives out the maximum light?
198. Ten bulbs are connected in a series circuit to a power supply line. Ten identical bulbs are
connected in a parallel circuit to an identical power supply line.
a) Which circuit would have the highest voltage across each bulb?
b) In which circuit would the bulbs be brighter?
c) In which circuit, if one bulb blows out, all others will stop glowing?
d) Which circuit would have less current in it?
199. Draw a circuit diagram showing two electric lamps connected in parallel together with a
cell, an ammeter and a switch that works both lamps.
200. Draw a circuit diagram to show how two 4 V electric lamps can be lit brightly from two 2 V
cells.
201. Consider the circuit given below where A, B and C are three identical light bulbs of constant
resistance.

a) List the bulbs in order of increasing brightness.


b) If C burns out, what will be the brightness of A now compared with before?
c) If B burns out instead, what will be the brightness of A and C compared with before?
202. The figure below shows a variable resistor in a dimmer switch. Explain, how would you turn
the switch to make the lights –

a) Brighter? b) Dimmer?
POWER
203. What is meant by electric power? Write all the 4 formulas for electric power.
204. Write down an equation linking Watts, Volts and Amperes.
205. Which quantity has the unit of watt?
206. Define the S.I. unit of electrical power.
207. Which one has a higher electrical resistance: a 100 W bulb or a 60 W bulb?
208. If the potential difference between the ends of a wire of fixed resistance is doubled, by
how much does the electrical power increase?
209. Define electrical energy.
210. State two factors on which the electrical energy consumed by an electrical appliance
depends.
211. State whether an electric heater will consume more electrical energy or less electrical
energy per second when the length of its heating element is reduced. Give reasons for your
answer.
212. What is the SI unit of electrical energy?
213. What is the commercial unit of electrical energy?
214. Name the quantity whose unit is –
a) Kilowatt. b) Kilowatt-hour.
215. What is the meaning of the symbol kWh? Which quantity does it represent?
216. Define watt-hour. How many joules are equal to 1 Watt-hour?
217. Define kilowatt-hour. How many joules are there in 1 kWh?
218. Three incandescent bulbs of 100 W each are connected in series in an electric circuit.
Another set of three bulbs of the same wattage are connected in parallel to the same
source. Will the bulb in the two circuits glow with the same brightness? Justify your
answer.
219. Two exactly similar electric lamps are arranged –
a) In parallel. b) In series.
If the parallel and series combinations of lamps are connected to 220 V supply line one by
one, what will be the ratio of electric power consumed by them?
220. An electric lamp is labelled 12 V, 36 W. This indicates that it should be used with a 12 V
supply. What other information does the label provide?
221. A bulb is rated as 250 V; 0.4 A. Find its power.
222. An electric bulb is connected to a 220 V power supply. If the bulb draws a current of 0.5 A,
calculate the power of the bulb.
223. What current will be taken by a 920 W appliance if the supply voltage is 230 V?
224. What is the maximum power in kilowatts of the appliance that can be connected safely to
13 A; 230 V mains socket?
225. An electric bulb is rated at 220 V, 100 W. What is its resistance?
226. The diagram below shows a circuit containing a lamp L, a voltmeter and an ammeter. The
voltmeter reading is 3 V and the ammeter reading is 0.5 A.

a) What is the resistance of the lamp?


b) What is the power of the lamp?
227. How much energy is consumed when a current of 5 A flows through the filament of a
heater having resistance of 100 ohms for 2 hours? Express it in joules.
228. In which of the following cases, more electrical energy is consumed per hour –
a) A current of 1 A passed through a resistance of 300 ohms.
b) A current of 2 A passed through a resistance of 100 ohms.
229. Which uses more energy: a 250 W TV set in 1 hour or a 1200 W toaster in 10 minutes?
230. An electric motor takes 5 Amperes current from a 220 Volt supply line. Calculate the power
of the motor and electrical energy consumed by it in 2 hours.
231. An electric kettle rated at 220 V, 2.2 kW, works for 3 hours. Find the energy consumed and
the current drawn.
232. The electric kettle connected to the 230 V mains supply draws a current of 10 A. Calculate
a) The power of the kettle. b) The energy transferred in 1 minute.
233. For a heater rated at 4 kW and 220 V, calculate –
a) The current.
b) The resistance of the heater.
c) The energy consumed in 2 hours.
d) The cost, if 1 kWh is priced at ₹ 4.60.
234. In a house, two 60 W electric bulbs are lighted for 4 hours, and three 100 W bulbs for 5
hours every day. Calculate the electrical energy consumed in 30 days.
235. Two lamps, one rated 40 W at 220 V and the other 60 W at 220 V, are connected in parallel
to the electric supply at 220 V.
a) Draw a circuit diagram to show the connections.
b) Calculate the current drawn from the electric supply.
c) Calculate the total energy consumed by the 2 lamps together when they operate for
1 hour.
236. Calculate the cost of operating a heater of 500 W for 20 hours at the rate of ₹ 3.90 per unit.
237. A boy noted the readings on his home’s electricity meter on Sunday at 8 AM and again on
Monday at 8 AM.

a) What was the meter reading on Sunday?


b) What was the meter reading on Monday?
c) How many units of electricity have been used?
d) In how much time these units have been used?
e) If the rate is ₹ 5 per unit, what is the cost of electricity used during this time?
238. A 2 kW heater, a 200 W TV and three 100 W lamps are all switched on from 6 pm to 10 pm.
What is the total cost at ₹ 5.50 per kWh?
239. An electric fan runs from the 230 V mains. The current flowing through it is 0.4 A. At what
rate is electrical energy transferred by the fan?
240. The table below shows the current in three different electric appliances when connected to
a 230 V mains supply.
Appliance Current
Kettle 8.5 A
Lamp 0.4 A
Toaster 4.8 A

a) Which appliance has the greatest electrical resistance? How does the data show this?
b) The lamp is connected to the mains supply by using a thin, twin-cored cable
consisting of live and neutral wires. State two reasons why this cable should not be
used for connecting the kettle to the mains supply?
c) Calculate the power rating of the kettle when it is operated from the 240 V mains
supply.
d) A man takes the kettle abroad where the mains supply is 120 V. What is the current
in the kettle when it is operated from the 120 V supply?
241. An electric bulb is rated 10 W, 220 V. How many of these bulbs can be connected in parallel
across the two wires on 220 V supply line if the maximum current which can be drawn is 5
A?
242. 𝐵1 , 𝐵2 and 𝐵3 are three identical bulbs connected as shown in figure. When all the three
bulbs glow, a current of 3 A is recorded by the ammeter A.

a) What will happen to the glow of the other two bulbs when the bulb 𝐵1 gets fused?
b) What will happen to the readings of 𝐴1 , 𝐴2 , 𝐴3 and 𝐴 when the bulb 𝐵2 gets fused?
c) How much power is dissipated in the circuit when all the three bulbs glow together?
243. Three 2 ohm resistors, A, B and C are connected as shown in figure. Each of them dissipates
energy and can with stand a maximum power of 18 W without melting. Find the maximum
current that can flow through the three resistors?
244. Calculate the power used in the 2 ohm resistor in each of the following circuits –
a) A 6 V battery in series with 1 ohm and 2 ohm resistors.
b) A 4 V battery in parallel with 12 ohm and 2 ohm resistors.
245. Find out the following in the electric circuit given in figure:

a) Effective resistance of two 8 Ω resistors in the combination.


b) Current flowing through 4 Ω resistor.
c) Potential difference across 4 Ω resistor.
d) Power dissipated in 4 Ω resistor.
e) Difference in ammeter readings, if any.

HEATING EFFECTS OF ELECTRIC CURRENT


246. What is meant by heating effect of current?
247. State and explain joule’s law of heating. How can it be demonstrated experimentally?
248. Write down the formula for the heat produced when a current 'I' is passed through a
resistor ‘R’ for time ‘t’.
249. Derive the expression for the heat produced due to a current 'I' flowing for a time period ‘t’
through a resistor ‘R’ having a potential difference ‘V’ across its ends. With which name is
this relation known?
250. State three factors on which the heat produced by an electric current depends. How does it
depend on these factors?
251. How does the heat ‘H’ produced by current passing through a fixed resistance wire depend
upon the magnitude of current ′𝐼′ ?
252. If the current passing a conductor is doubled, what will be the change in the heat
produced?
253. The current passing through a room heater has been halved. What will happen to the heat
produced by it?
254. Two exactly similar heating resistances are connected in series and in parallel in two
different circuits one by one. If the same current is passed through both the
combinations, is more heat obtained per minute when they are connected in series or
when they are connected in parallel? Give reason for your answer.
255. Explain any three applications of the heating effect of current.
256. Explain why tungsten is used for making the filament of an electric bulb.
257. Which gas is filled in an electric bulb and why?
258. Why does the connecting cord of an electric heater not glow hot while the heating element
does?
259. Explain why, the current that makes the heater element very hot, only slightly warms the
connecting wires leading to the heater.
260. The electrical resistivities of P, Q, R and S are given below:
P 6.84 × 𝟏𝟎−𝟖 𝛀𝐦
Q 1.70 × 𝟏𝟎−𝟖 𝛀𝐦
R 1.0 × 𝟏𝟎𝟏𝟓 𝛀𝐦
S 11.0 × 𝟏𝟎−𝟕 𝛀𝐦
Which material will you use for making –
a) Heating element of electric iron?
b) Connecting wires of electric iron?
c) Covering of connecting wires?
Give reason for your choice in each case.
261. Name the material which is used for making the filaments of an electric bulb.
262. Explain why, tungsten is used almost exclusively for the filaments of electric lamps?
263. How does the wire in the filament of a light bulb behave differently to the other wires in
the circuit when the current flows? What property of the filament wire accounts for this
difference?
264. Name two gases which are filled in filament type electric light bulbs.
265. Explain why argon or nitrogen is filled in an electric bulb but not with air.
266. Explain why, filament type electric bulbs are not power efficient?
267. Name two effects produced by electric current.
268. What effect of current is utilised in an electric heater, electric iron, electric geyser, etc.?
269. What effect of current is utilised in an electric bulb?
270. What effect of current is utilised in the working of an electric fuse?
271. Name three devices which work on the heating effect of electric current.
272. Which electric heating devices in your home do you think have resistors which control the
flow of electricity?
273. An electric iron of resistance 20 ohms draws a current of 5 A. Calculate the heat produced
in 30 seconds.
274. A resistance of 40 ohms and one of 60 ohms are arranged in series across 220 V supply.
Find the heat in joules produced by this combination of resistances in half a minute.
275. 100 joules of heat is produced per second in a 4 ohm resistor. What is the potential
difference across the resistor?
276. Compute the heat generated while transferring 96000 coulombs of charge in one hour
through a potential difference of 50 V.
277. When a current of 4 A passes through a certain resistor for 10 minutes, 2.88 × 104 J of heat
are produced. Calculate –
a) The power of the resistor. b) The voltage across the resistor.
278. A heating coil has a resistance of 200 ohms. At what rate will heat be produced in it when a
current of 2.5 A flows through it?
279. An electric heater of resistance 8 ohms takes a current of 15 A from the mains supply line.
Calculate the rate at which heat is developed in the heater?
280. A resistance of 25 ohms is connected to a 12 V battery. Calculate the heat energy in joules
generated per minute.
281. How much heat will an instrument of 12 W produced in 1 minute if it is connected to a
battery of 12 V?
282. An electric iron is connected to the mains power supply of 220 V. when the electric iron is
adjusted at minimum heating, it consumes a power of 360 W but at ‘maximum heating’ it
takes a power of 840 W. Calculate the current and resistance in each case.
MULTIPLE CHOICE QUESTIONS
283. The proper representation of series combination of cells obtaining maximum potential is

a) (i) b) (ii)
c) (iii) d) (iv)
284. One coulomb charge is equivalent to the charge contained in:
a) 2.6 × 1019 electrons b) 6.2 × 1019 electrons
c) 2.65 × 1018 electrons d) 6.25 × 1018 electrons
285. How many electrons constitute a current of one micro-ampere?
a) 6.25 × 106 electrons b) 6.25 × 1012 electrons
c) 6.25 × 109 electrons d) 6.25 × 1015 electrons
286. The work done in moving a unit charge across two points in an electric circuit is a measure
of:
a) Current b) Potential difference
c) Resistance d) Power
287. Which of the following represents voltage?
𝑊𝑜𝑟𝑘 𝑑𝑜𝑛𝑒 b) 𝑊𝑜𝑟𝑘 𝑑𝑜𝑛𝑒 × 𝐶ℎ𝑎𝑟𝑔𝑒
a) 𝐶𝑢𝑟𝑟𝑒𝑛𝑡 × 𝑡𝑖𝑚𝑒
𝑊𝑜𝑟𝑘 𝑑𝑜𝑛𝑒 ×𝑡𝑖𝑚𝑒 d) 𝑊𝑜𝑟𝑘 𝑑𝑜𝑛𝑒 × 𝐶ℎ𝑎𝑟𝑔𝑒 × 𝑡𝑖𝑚𝑒
c) 𝐶𝑢𝑟𝑟𝑒𝑛𝑡

288. The unit for measuring potential difference is:


a) Watt b) Ohm
c) Volt d) Coulomb
289. The device used for measuring potential difference is known as:
a) Potentiometer b) Ammeter
c) Galvanometer d) Voltmeter
290. The other name of potential difference is:
a) Ampereage b) Wattage
c) Voltage d) Potential energy
291. Which unit could be used to measure current?
a) Watt b) Coulomb
c) Volt d) Ampere
292. Electric current is measured by
a) A voltmeter b) An ammeter
c) A potentiometer d) An anemometer
293. If the current through a flood lamp is 5 A, what charge passes in 10 s?
a) 0.5 C b) 2C
c) 5C d) 50 C
294. If the amount of electric charge passing through a conductor in 10 minutes is 300 C, the
current flowing is:
a) 30 A b) 0.3 A
c) 0.5 A d) 5A
295. A current of 1 A is drawn by a filament of an electric bulb. Number of electrons passing
through a cross section of the filament in 16 𝑠 would be roughly –
a) 1020 b) 1016
c) 1018 d) 1023
296. Ohm’s law gives a relation between
a) Current and resistance
b) Resistance and potential difference
c) Potential difference and electrical charge
d) Current and potential difference
297. The unit of electrical resistance is
a) Ampere b) Volt
c) Coulomb d) Ohm
298. Keeping the potential difference constant, the resistance of a circuit is doubled. The current
will become
a) Double b) Half
c) One-fourth d) Four times
299. Keeping the p.d. constant, the resistance of a circuit is halved. The current will become
a) One-fourth b) Four times
c) Half d) Double
300. A student carries out an experiment and plot the V-I graphs of three samples of nichrome
wire with resistances 𝑅1 , 𝑅2 and 𝑅3 respectively, as shown in the figure. Which of the
following is true?

a) 𝑅1 = 𝑅2 = 𝑅3 b) 𝑅1 > 𝑅2 > 𝑅3
c) 𝑅3 > 𝑅2 > 𝑅1 d) 𝑅2 > 𝑅3 > 𝑅1
301. The p.d. across a 3 Ω resistor is 6 V. The current flowing in the resistor will be
1
a) A b) 1A
2

c) 2A d) 6A
302. A car headlight bulb working on a 12 V car battery draws a current of 0.5 A. The resistance
of the light bulb is
a) 0.5 Ω b) 6Ω
c) 12 Ω d) 24 Ω
303. An electrical appliance has a resistance of 25 Ω. When this electrical appliance is connected
to a 230 V supply line, the current passing through it will be
a) 0.92 A b) 2.9 A
c) 9.2 A d) 92 A
304. When a 4 Ω resistor is connected across the terminals of a 12 V battery, the number of
coulombs passing through the resistor per second is
a) 0.3 b) 3
c) 4 d) 12
305. The substance having infinitely high resistance is called
a) Conductor b) Resistor
c) Superconductor d) Insulator
306. The resistivity of a certain material is 0.6 Ωm. The material is most likely to be
a) An insulator b) A superconductor
c) A conductor d) A semiconductor
307. The unit of resistivity is
a) Ohm b) Ohm metre2
c) Ohm-meter d) Ohm per meter
308. Electrical resistivity of a given metallic wire depends upon –
a) Its length. b) Its thickness.
c) Its shape. d) Nature of the material.
309. The resistivity of copper metal depends on only one of the following factors. The factor is
a) Length b) Thickness
c) Temperature d) Area of cross-section
310. The resistivity does not change if –
a) The material is changed.
b) The temperature is changed.
c) The shape of the resistor is changed.
d) Both material and temperature are changed.
311. If the area of cross-section of a resistance wire is halved, then its resistance becomes
a) Half b) Two times
c) One-fourth d) Four times
312. When the area of cross-section of a conductor is doubled, its resistance becomes
a) Double b) Half
c) Four times d) One-fourth
313. If the diameter of resistance wire is half, then its resistance becomes
a) Four times b) Half
c) One-fourth d) Two times
314. When the diameter of a wire is doubled, its resistance becomes
a) Double b) Four times
c) One-half d) One-fourth
315. A cylindrical conductor of length 𝒍 and uniform area of cross-section A has resistance 𝑹.
Another conductor of length 𝟐𝒍 and resistance R of the same material has area of cross-
section –
A 3A
a) b)
2 2

c) 2A d) 3A
316. The resistance of a wire of length 300 𝑚 and cross section area 1.0 𝑚𝑚2 made of material
of resistivity 1.0× 10−7 Ωm is
a) 2Ω b) 3Ω
c) 20 Ω d) 30 Ω
317. If the resistance of a certain copper wire is 1 Ω, then the resistance of a similar nichrome
wire will be about
a) 25 Ω b) 30 Ω
c) 60 Ω d) 45 Ω
1
318. What is the maximum resistance which can be made using 5 resistors each of Ω.
5
1
a) Ω b) 10 Ω
5

c) 5Ω d) 1Ω
1
319. What is the minimum resistance which can be made using 5 resistors each of Ω.
5
1 1
a) Ω b) Ω
5 25
1
c) Ω d) 25 Ω
10

320. The circuit in which the electrical components have been properly connected.

a) (i) b) (ii)
c) (iii) d) (iv)
321. A cell, a resistor, a key and an ammeter are arranged as shown in the circuit diagram of
figure. The current recorded in the ammeter will be –

a) Maximum in (i). b) Maximum in (ii).


c) Maximum in (iii). d) Same in all the cases.
322. Two resistors of resistance 2 Ω and 4 Ω when connected to a battery will have
a) Same current flowing through them when connected in parallel.
b) Same current flowing through them when connected in series.
c) Same potential difference across them when connected in series.
d) Different potential difference across them when connected in parallel.
323. Which statement is correct?
a) An ammeter is connected in series in a circuit and a voltmeter is connected in
parallel.
b) An ammeter has a high resistance.
c) A voltmeter has a low resistance.
d) None of these.
324. A battery and three lamps are connected as shown below. Which of the following
statements about the currents at X, Y and Z is correct?
a) The current at Z is greater than that at Y.
b) The current at Y is greater than that at Z.
c) The current at X equals the current at Y.
d) The current at X equals the current at Z.
325. The figure given below shows three resistors. Their combined resistance is:

5
a) 17 Ω b) 14 Ω
2
c) 63 Ω d) 40 Ω

326. The diagram below shows part of a circuit. In this arrangement of three resistors was to be
replaced by a single resistor, its resistance should be:

a) 9Ω b) 4Ω
c) 6Ω d) 18 Ω
327. If two resistors of 25 Ω and 15 Ω are joined together in series and then placed in parallel
with a 40 Ω resistor, the effective resistance of the combination is:
a) 0.1 Ω b) 10 Ω
c) 20 Ω d) 40 Ω
328. In the circuit shown below, the potential difference across the 3 Ω resistor is:

1 1
a) V b) V
9 2

c) 1V d) 2V
329. V1 , V2 and V3 are the p.ds. across the 1 Ω, 2 Ω and 3 Ω resistors in the following diagram,
and the current is 5A. Which one of the columns (a) to (d) shows the correct values of V1 ,
V2 and V3 measured in volts?

a) V1 =1.0, V2 =2.0, V3 =3.0 b) V1 =5.0, V2 =10.0, V3 =15.0


c) V1 =5.0, V2 =2.5, V3 =1.6 d) V1 =4.0, V2 =3.0, V3 =2.0
330. The lamps in a household circuit are connected in parallel because –
a) this way they require less current.
b) if one lamp fails the others remain lit.
c) this way they require less power.
d) if one lamp fails, the others also fail.
331. Using the circuit given below, state which of the following statement is correct?

a) When 𝑆1 and 𝑆2 are closed, lamps A and B are lit.


b) With 𝑆1 open and 𝑆2 closed, A is lit and B is not lit.
c) With 𝑆2 open and 𝑆1 closed, A and B are lit.
d) With 𝑆1 closed and 𝑆2 open, lamp A remains lit even if lamp B gets fused.
332. Which of the following terms does not represent electrical power in a circuit?
a) 𝐼2𝑅 b) 𝐼𝑅 2
c) 𝑉𝐼 d) 𝑉 2 ⁄𝑅
333. Unit of electric power may also be expressed as
a) Volt ampere b) Kilowatt hour
c) Watt second d) Joule second
334. The unit for expressing electric power is:
a) Volt b) Joule
c) Coulomb d) watt
335. When an electric lamp is connected to 12 V battery, it draws a current of 0.5 A. The power
of the lamp is:
a) 0.5 W b) 6W
c) 12 W d) 24 W
336. An electric heater is rated at 2 kW. Electrical energy costs ₹ 4 per kWh. What is the cost of
using the heater?
a) ₹ 12 b) ₹ 24
c) ₹ 36 d) ₹ 48
337. The S.I. unit of energy is:
a) Joule b) Coulomb
c) Watt d) Ohm-meter
338. The commercial unit of energy is:
a) Watt b) Watt-hour
c) Kilowatt-hour d) Kilo-joule
339. How much energy does a 100 W electric bulb transfer in 1 minute?
a) 100 J b) 600 J
c) 3600 J d) 6000 J
340. An electric kettle for use on a 230 V supply is rated at 3000 W. For safe working, the cable
connected to it should be able to carry at least:
a) 2A b) 5A
c) 10 A d) 15 A
341. How many joules of electrical energy are transferred per second by a 6 V; 0.5 A lamp?
a) 30 J/s b) 12 J/s
c) 0.83 J/s d) 3 J/s
342. At a given time, a house is supplied with 100 A at 220 V. How many 75 W, 220 V light bulbs
could be switched on in the house at the same time (if they are all connected in parallel)?
a) 93 b) 193
c) 293 d) 393
343. If the potential difference between the ends of a fixed resistor is halved, the electrical
power will become:
a) Double b) Half
c) Four times d) One-fourth
344. Which of the following is likely to be the correct wattage for an electric iron used in our
homes?
a) 60 W b) 250 W
c) 850 W d) 2000 W
345. If the current 'I' through a resistor is increases by 100 % (assume that temperature remains
unchanged), the increase in power dissipated will be
a) 100 % b) 200 %
c) 300 % d) 400 %
346. In a filament type light bulb, most of the electric power consumed appears as:
a) Visible light b) Infra-red rays
c) Ultraviolet rays d) Fluorescent light
347. In an electrical circuit, three incandescent bulbs A, B and C of rating 40 W, 60 W and 100 W,
respectively are connected in parallel to an electric source. Which of the following is likely
to happen regarding their brightness?
a) Brightness of all the bulbs will be the same.
b) Brightness of bulb A will be the maximum.
c) Brightness of bulb B will be more than that of A.
d) Brightness of bulb C will be less than that of B.
348. An electric kettle consumes 1 kW of electric power when operated at 220 V. A fuse wire of
what rating must be used for it
a) 1A b) 2A
c) 4A d) 5A
349. The heat produced in a wire of resistance ′𝑥′ when a current ′𝑦′ flows through it in
time ′𝑧′ is given by:
a) 𝑥2 × 𝑦 × 𝑧 b) 𝑥 × 𝑧 × 𝑦2
c) 𝑦 × 𝑧2 × 𝑥 d) 𝑦×𝑧×𝑥
350. The heat produced by passing an electric current through a fixed resistor is proportional to
the square of
a) Magnitude of resistance of the resistor.
b) Temperature of the resistor.
c) Magnitude of current.
d) Time for which current is passed.
351. The current passing through an electric kettle has been doubled. The heat produced will
become:
a) Half b) Double
c) Four times d) One-fourth
352. If the current flowing through a fixed resistor is halved, the heat produced in it will
become:
a) Double b) One-half
c) One-fourth d) Four times
353. In an electrical circuit, two resistors of 2 Ω and 4 Ω respectively are connected in series to a
6 V battery. The heat dissipated by the 4 Ω resistor in 5 s will be
a) 5J b) 10 J
c) 20 J d) 30 J
354. In the following circuits, heat produced in the resistor or combination of resistors
connected to a 12 V battery will be

a) Same in all the cases b) Maximum in case (i)


c) Maximum in case (ii) d) Maximum in case (iii)
355. Two conducting wires of the same material and of equal lengths and equal diameters are
first connected in series and then in parallel in a circuit across the same potential
difference. The ratio of heat produced in series and parallel combination would be:
a) 1:2 b) 2:1
c) 1:4 d) 4:1
356. An electric fuse works on the:
a) Chemical effect of current. b) Magnetic effect of current.
c) Lighting effect of current. d) Heating effect of current.
357. The elements of electrical heating devices are usually made of:
a) Tungsten b) Bronze
c) Nichrome d) Argon
358. ______________ is used exclusively for the filaments of the bulbs.
a) Copper b) Aluminium
c) Silver d) tungsten
359. The commonly used safety fuse wire is made of
a) Copper b) lead
c) nickel d) an alloy of tin and copper
360. Which of the following characteristics is not suitable for a fuse wire?
a) Thin and short b) Thick and short
c) Low melting point d) Higher resistance than rest of wiring
361. Which of the following is the most likely temperature of the filament of an electric light
bulb when it is working on the normal 220 V supply line?
a) 500° C b) 1500° C
c) 2500° C d) 4500° C
ASSERTION – REASON
The following questions consists of two statements – Assertion (𝐴) and Reason (𝑅). Answer
these questions selecting the appropriate option given below:

a) Both 𝐴 and 𝑅 are true and 𝑅 is the correct explanation for 𝐴.


b) Both 𝐴 and 𝑅 are true and 𝑅 is not the correct explanation for 𝐴.
c) 𝐴 is true but 𝑅 is false.
d) 𝐴 is false but 𝑅 is true.
362. Assertion (A): Tungsten metal is used for making filaments of incandescent lamps.
Reason (R): The melting point of tungsten is very low.
363. Assertion (A): If a graph is plotted between potential difference and the current flowing,
the graph is a straight line passing through the origin.
Reason (R): The current is directly proportional to the potential difference.
364. Assertion (A): Longer wires have greater resistance and the smaller wires have lesser
resistance.
Reason (R): Resistance is inversely proportional to the length of the wire.
365. Assertion (A): Alloys are commonly used in electrical heating devices like electric iron,
toasters, etc.
Reason (R): Alloys do not oxidise (burn) readily at high temperatures.
366. Assertion (A): The value of the current in the ammeter is the same, independent of its
position in the electric circuit.
Reason (R): In a series combination of resistors, the current is the same in every part of the
circuit or the same current through each resistor.
367. Assertion (A): When the resistors are connected end to end consecutively, they are said to
be in series.
Reason (R): In case the total resistance is to be increased, then the individual resistances
are connected in series.
368. Assertion (A): When the resistances are connected between the same two points, they are
said to be connected in parallel.
Reason (R): In case the resistance is to be decreased, then the individual resistances are
connected in parallel.
369. Assertion (A): A cell is a device which converts chemical energy into electrical energy.
Reason (R): A cell maintains a constant potential difference between its terminals for a long
time.

TRUE – FALSE
370. When two resistances 1 Ω and 3 Ω are connected in parallel, their equivalent resistance is
less than 1 Ω.
371. The resistivity of a wire is directly proportional to cross-sectional area.
372. In the circuit to verify ohm’s law, ammeter and voltmeter both are connected in series with
resistance and cell in the circuit.
373. Two wires of resistances 2 Ω and 4 Ω are connected in series. The combination is connected
to a 220 V supply. The power dissipated in 2 Ω resistor is more.
374. Two wires of resistances 2 Ω and 4 Ω are connected in parallel. The combination is
connected to a 220 V supply. The power dissipated in 2 Ω resistor is more.

FILL IN THE BLANKS


375. Potential difference is measured in __________ by using a _______________ placed in
_______________ across a component.
376. Copper is a good ___________________. Plastic is an___________________.
377. A current is a flow of _____________. For this to happen there must be a ________ circuit.
378. Current is measured in ____________ using an_____________ placed in ____________ in
a circuit.
379. Ohm’s law states a relation between potential difference and _______________.
380. Resistance is measured in __________. The resistance of a wire increases as the length
_____________; as the temperature _____________; and as the cross sectional area
______________.
ANSWERS
1) A charge is a physical entity which is defined by excess or deficiency of electrons on a body.
A body is said to be negatively charged if it gains electrons. E.g. an ebonite rod rubbed with
fur acquires negative charge. A body is said to be positively charged if it loses electrons. E.g.
a glass rod rubbed with a silk cloth acquires positive charge.
2) Coulomb (C).
3) One coulomb is that quantity of electric charge which is equal to the amount of charge
transferred by a current of 1 ampere in 1 second.
4) Conductors: Those substances through which electricity can flow are known as conductors.
E.g. copper, silver etc.
Insulators: Those substances through which electricity cannot flow are known as insulators.
E.g. plastic, cotton etc.
5) Conductors: silver, copper, aluminium, nichrome, graphite, mercury, manganin.
Insulators: sulphur, cotton, air, paper, porcelain, mica, Bakelite, polythene.
6) The electric potential (or potential) at a point in an electric field is defined as the work done
in moving a unit positive charge from infinity to that point. Unit of electric potential is volt
(V).
7) Potential difference between two points in an electric circuit is defined as the amount of
work done in moving a unit charge from one point to the other point.
8) Electric potential at a point is 1 volt means 1 joule of work is done in moving 1 unit positive
charge from infinity to that point.
9) The potential difference between two points is 1 volt means 1 joule of work is done in
moving 1 coulomb of electric charge from one point to the other.
10) Volt (V).
Work done W
11) Potential difference = . i.e. V = .
Charge moved Q

12) Volt (V).


13) 1 joule (J).
14) Cell or Battery.
15) Voltmeter.
16) A voltmeter is a device which is used to measure the potential difference between two
points in an electric circuit. Voltmeter is always connected in parallel across the two points
where the potential difference is to be measured.

17) Copper has free electrons that are loosely held by the nuclei of the atoms. These free
electrons result in conduction of electricity. The electrons present in rubber are strongly
held by the nuclei of its atoms. So, rubber does not have free electrons to conduct
electricity.
18) 6.25 × 1018 electrons
19) 24 J
20) 12.5 V
21) 20 J
22) 40 J
23) 6J
24) 12 J
25)
a) 6V
b)
i) 2J ii) 6J
26) Electric current.
27) An electric current is a flow of electric charges (electrons) through a conductor. Potential
difference between the ends of the wire makes electric current to flow in the wire.
28) Conventional current flows from positive terminal of a battery to the negative terminal,
through the outer circuit.
29) Electrons flow from negative terminal to positive terminal of the battery (opposite to the
direction of conventional current).
30) Electrons.
31) Electrons.
Charge moved Q
32) Electric current = . i.e. 𝐼 = .
Time taken t

33) S.I. Unit of electric current is Ampere (A). When 1 coulomb of charge flows through any
cross-section of a conductor in 1 second, the electric current flowing through it is said to be
1 ampere.
34) Ampere (A).
35) 1 ampere (A).
36) 1 ampere = 1000 milli ampere. i.e. 1 A = 103 mA
37) 1 ampere = 1000000 micro ampere. i.e. 1 A = 106 μA
38) Ammeter.
39) Ammeter is a device used for the measurement of electric current. It is always connected in
series with the circuit in which the current is to be measured.

40) The resistance of an ammeter should be very small so that it may not change the value of
the current flowing in the circuit. An ammeter has to measure the current flowing through
the circuit whereas resistance offers an obstruction to the current flow. So, if the resistance
of an ammeter is large, the current measured by the ammeter will be quite less as
compared to the actual amount of current flowing through the circuit, which is undesirable.
An ideal ammeter should have zero resistance so that it can give exact value of current, but
this is not possible because every material has some value of internal resistance which we
can’t control.

Voltmeters are put in parallel with the load in order to measure the potential difference
between two different points. If they are in parallel with the load, then at the first junction,
the current will split and flow into both paths (Kirchhoff’s law). However, if current is
flowing through the voltmeter, then it is not all flowing through the load, and the potential
difference across the load would change when the voltmeter is added and removed, which
is unfavourable. An ideal voltmeter should have infinite resistance so that current does not
flow through it at all, but this is not possible.
41) A continuous conducting path consisting of wires and other resistances (like electric bulb,
etc) and a switch, between the two terminals of a cell or battery, along which an electric
current flows, is called a circuit.
42)

43)

44)
a) The lamps are in series.
b) Student has connected ammeter in parallel with lamps. It should be connected in
series.
c)
45)

Errors Corrections

Cells are not connected in series Cells must be connected in series and that
properly. properly too.

Ammeter is connected in parallel with R. It should be connected in series with R.

Voltmeter connected in series with R. It connected in parallel with R.

Signs of the terminals of voltmeter are Signs of the terminals of voltmeter need to
not mentioned and that of ammeter are be mentioned and that of ammeter must be
not correct. corrected.

Current is coming out from negative


Direction of current and polarity of cell
terminal and going into negative terminal
needs to be corrected.
again.

Correct circuit diagram:

46)

47) 20 A
48) 0.5 A
49) 300 C
50) 40 C
51) 324 C
52) 8 V and 2.5 A
53) 1000 A and 100 MJ (or 108 J)
54) 6.25 × 1018 electrons
55) 31.25 × 1018 electrons
56)
a) 12 J b) 60 J c) 240 J
57) 1200 J
58)
a) 8C b) 9200 J
59) Ohm’s law.
60) Ohm’s law gives a relationship between current (𝐼) and potential difference (𝑉).
According to ohm’s law: At constant temperature, the current flowing through a conductor
is directly proportional to the potential difference across its ends.
If '𝐼' is the current flowing through a conductor and ′𝑉′ is the p.d. across its ends, then
According to the ohm’s law:
𝐼∝V
or V ∝ 𝐼
or, V = 𝐼R
V
or, R =
𝐼
Where, ‘R’ is the constant of proportionality, called “Resistance” of the conductor.
The unit of resistance is ohm (Ω).
61) The experimental set up of ohm’s law is made as the circuit shown below, consisting of a
nichrome wire XY, an ammeter (A). A voltmeter (V), a key and four cells of 1.5 V each.

The reading in the ammeter A for the current and reading of the voltmeter V for the
potential difference across the nichrome wire XY is measured and recorded in the table
given below, first by using only one cell and then by the two, three and four cells in
successive readings.
S. No Number of Potential difference Current through the V⁄𝑰
cells used in across the nichrome wire, nichrome wire, (volt/ampere)
the circuit 𝑽 (volt) 𝑰 (ampere)
1 1 1.5 V 0.1 A 15 Ω
2 2 3.0 V 0.2 A 15 Ω
3 3 4.5 V 0.3 A 15 Ω
4 4 6.0 V 0.4 A 15 Ω

Calculating the ratio of V to 𝐼 for each pair of potential difference V and current 𝐼. A graph
is plotted between V and 𝐼 and the nature of the graph is observed as shown here.

The same value for 𝑉⁄𝐼 is obtained corresponding to one, two, three and four cells in
successive readings. Also, the 𝑉 − 𝐼 graph is obtained as straight line that passes through
the origin. This law is only valid for ohmic conductor. E.g. metals.
Thus, 𝑉 ⁄𝐼 is constant i.e. 𝑉 ∝ 𝐼
This verifies ohms law.
Ohm’s law does not hold good under all conditions as it is not a fundamental law of nature
like Newton’s law. It is obeyed by metallic conductors only when physical conditions like
temperature, etc are kept unchanged. It is not obeyed by a lamp filament, junction diode,
thermistor, etc. as these are non-ohmic conductors.
62) 1 ohm is the resistance of a conductor such that when a potential difference of 1 volt is
applied to its ends, a current of 1 ampere flows through it.
63) Resistance.
64) The property of a conductor due to which it opposes the flow of current through it is called
resistance of the conductor. Potential difference = Current × Resistance. i.e. V=𝐼𝑅.
65) Ohm. Its symbol is Ω.
66) Electric resistance.
67) 3.4 Ω.

68)
a) Ohm’s law.
b) Temperature.
69) Since the graph is a straight line passing through the origin, so current is directly
𝑉
proportional to the potential difference. Hence, the ratios of 𝐼
remains constant.
𝑉 1.5
From graph, when V = 1.5 volt and 𝐼 = 0.6 amp, then R = = = 2.5 ohms.
𝐼 0.6
𝑉
So, for p.d. 0.8 V, 1.2 V and 1.6 V, the value of remains the same. i.e. 2.5 ohms.
𝐼
70) Strength of electric current flowing in a given conductor depends on –
a) potential difference across the ends of the conductor
b) resistance of the conductor
71) The current will also decrease to half of its former value as current is directly proportional
to potential difference.
72) The current will increase to double of its former value as current is inversely proportional
to resistance.
73) The current will decrease to half of its former value as current is inversely proportional to
resistance.
74) Insulators.
75) Conductors: Those substances which have very low electrical resistance are called as good
conductors. E.g. Copper and Aluminium.
Resistors: Those substances which have comparatively high resistance than conductors are
known as resistors. E.g. Nichrome and Manganin.
Insulators: Those substances which have infinitely high electrical resistance are called
insulators. E.g. Rubber and Wood.
76) Conductors: Mercury, Aluminium, Iron, Metal coin.
Resistors: Manganin, Nichrome.
Insulators: Rubber, Polythene, Wood, Bakelite, Paper, Thermocol.
77) Electricians wear rubber hand gloves while working with electricity because rubber is an
insulator and protects them from electric shocks.
78) Silver metal is the best conductor of electricity because it has the lowest electrical
resistivity (1.60 × 10−8 Ω𝑚).
79) 0.05 Ω
80) 0.4 A
81) 1.7 A
82) 4A
83) 40 V
84) 0.6 Ω
85) 4800 Ω
86) 240 V
87) 48 Ω
88) 4.8 A
89) 125 V
90) 800 V
91) The property of a conductor due to which it opposes the flow of current through it is called
resistance of the conductor. Resistance of a conductor depends on the following factors –
a) Length of the conductor.
b) Area of cross-section (thickness or diameter) of the conductor.
c) Temperature of the conductor.
d) Nature of material of the conductor.
92)
a) Resistance of a conductor is directly proportional to the length of the conductor. i.e.
if length increases then resistance also increases or vice-versa.
b) Resistance of a conductor is inversely proportional to the area of cross-section of the
conductor. i.e. if area of cross-section increases then resistance decreases or vice-
versa.
c) Resistance of all pure metals is directly proportional to the temperature of the
conductor. i.e. if temperature increases then resistance also increases or vice-versa.
But the resistance of alloys like manganin, constantan and nichrome is almost
unaffected by temperature.
d) Some materials have naturally low resistance and some have high. E.g. if we take two
similar wires, having equal lengths and diameters, of copper metal and nichrome
alloy, we will find that the resistance of nichrome wire is about 60 times more than
that of the copper wire. This shows that the resistance of a conductor depends on
the nature of the material of the conductor.
93) If we take two similar wires of same length and same diameter, one of copper metal and
other of nichrome alloy, we will find that the resistance of nichrome wire is about 60 times
more than that of the copper wire. This shows that the resistance depends on the nature of
material of the conductor.
94)
a) If the length increases then the resistance also increases because resistance of a
conductor is directly proportional to the length of the conductor.
b) If the diameter increases then the resistance decreases because resistance of a
conductor is inversely proportional to the diameter of the conductor.
c) If the temperature increases then resistance also increases because resistance of a
conductor is directly proportional to the temperature of the conductor.
95) The resistance increases as resistance is inversely proportional to the thickness.
96) The resistance decreases as resistance is inversely proportional to the thickness.
97) Thick wire.
98) Current will flow more easily through thick wire because the resistance of the thick wire
will be lesser than that of thin wire.
99) Resistance also gets doubled.
100) Consider an electric circuit consisting of a cell, an ammeter, a nichrome wire of length L
(marked 1 to 4) and a plug key as shown below.

At point plug the key and note the current in ammeter. Replace the nichrome wire by
another nichrome wire as same thickness but twice the length .i.e. 2L at point 2. Again note
the reading. Now, replace the wire by thicker nichrome wire of same length L (marked 3). A
thicker wire has a larger cross- sectional area. Again note down the current through the
circuit. Replace nichrome wire with copper wire of same length and same area of cross-
section at point 4. Note the value of current. Notice the difference in the circuit in all cases.
When the length of the wire is doubled, then the ammeter reading decreases to half. Its
previous value .i.e. current through the wire is halved. Since, resistance of the wire R is
𝑉
equal to , then R is doubled which implies 𝑅 ∝ 𝐿. When the nichrome wire is replaced by
𝐼

a thicker one of same material and length, the current in the wire increases which means
that the resistance of the thicker wire (3) is less than that of thinner wire (1). This
1
implies 𝑅 ∝ . When the nichrome wire is replaced by the copper wire (4) of the same
𝐴

length and same cross-sectional area, then the current recorded by the ammeter is more.
This means that the resistance of copper wire is less than that of the nichrome wire of the
same dimensions. i.e. the resistance of the wire depends on the nature of its material.
101) Resistivity is the characteristic property of a substance which depends on the nature of the
substance and its temperature.
Resistance (R) × Area (A) RA
Resistivity (ρ) = . i.e. ρ =
Length (𝑙) 𝑙
Resistivity (ρ) × Length (𝑙) ρ𝑙
102) Resistance (R) = . i.e. R =
Area (A) A

103) Ohm-meter (Ωm).


104) Resistivity of a substance depends on the nature of the substance and its temperature. It
does not depend on the length or thickness of the conductor.
105)
Resistance Resistivity
Resistance is the property of the Resistivity is the property of the
conductor. material of the conductor.
Resistance of a conductor is the Resistivity of a substance is the
opposition to the flow of electric current opposition to the flow of electric
through it. current by a rod of that substance
which is 1 𝑚 long and 1 𝑚2 in cross-
section.
Resistance of a conductor depends on Resistivity of a substance depends on
length, thickness, nature of material and the nature of the substance and
temperature of the conductor. temperature.

106) On decreasing the temperature, the resistance decreases.


107)
a) When 𝑙 → 3𝑙
ρ𝑙
R=
A
ρ3𝑙 ρ𝑙
R' = = 3 = 3𝑅
A A
Resistance gets tripled.
b) When 𝑑 → 3𝑑
ρ𝑙
R=
A
ρ𝑙 ρ𝑙
R= =
𝜋𝑟 2 𝑑 2
𝜋 (2)
ρ𝑙 1 ρ𝑙 1 ρ𝑙 1 ρ𝑙 𝑅
R' = = = = =
3𝑑 2 9 𝑑 2 9 𝜋𝑟 2 9 A 9
𝜋( 2 ) 𝜋 (2 )

Resistance becomes one-ninth.


c) When 𝜌 → 3𝜌
ρ𝑙
R=
A
3ρ𝑙 ρ𝑙
R' = = 3 = 3𝑅
A A
Resistance becomes 3 times.
108)
a) Material ‘Q’ can be used for making electric wires because it has very low resistivity.
b) Material ‘R’ can be used for making handle of soldering iron because it has very high
resistivity.
c) Material ‘P’ can be used for making solar cells because it is a semi-conductor.
109)
a) Good conductor: C b) Resistor: A
c) Insulator: B d) Semi-Conductor: D
110)
a) E is the best conductor of electricity due to its least electrical resistivity.
b) C, because its resistivity is lesser than that of A.
c) B, because it has the highest electrical resistivity.
d) C and E, because of their low electrical resistivities.
111) Silver and copper are good conductors of electricity because they have free electrons
available for conduction and their resistivities are very low.
112) Iron.
113) Copper and aluminium wires are usually used for transmission of electricity because copper
and aluminium have very low electrical resistivity due to which they are very good
conductors of electricity.
114) Nichrome.
115) Nichrome is an alloy of nickel, chromium, manganese and iron having a resistivity of about
60 times more than that of copper. It is used for making the heating elements of electrical
heating appliances.
116) Presence of impurities in a metal increases its resistance.
117) Nichrome alloy is used to make the heating element of electrical appliances because –
a) Nichrome has very high resistivity.
b) Nichrome doesn’t go under oxidation easily even at high temperature.
118) Instead of a pure metal, an alloy is used to make the coils of electric irons and electric
toasters because –
a) Resistivity of an alloy is much higher than that of a pure metal.
b) An alloy doesn’t go under oxidation (or burn) easily even at high temperature, when
it is red hot.
119) 1.7 × 10−8 Ωm
120)
a) 122.7 𝑚 b) The resistance will become one-fourth.
121) 1.5 Ω
122) 31.4 × 10−8 Ωm
123) 0.036 Ω
124) 86.5 Ω
125) 86 Ω
126) 8.0 × 10−8 𝑚2
127) 1.84 × 10−6 Ωm
128) 80 Ω
129) According to the law of combination of resistances in series, the combined resistance of
any number of resistances connected in series is equal to the sum of the individual
resistances.
130) According to the law of combination of resistances in parallel, the reciprocal of the
combined resistance of a number of resistances connected in parallel is equal to the sum of
the reciprocals of all the individual resistances.
131) Let the current in the circuit be ′𝐼′ amperes and the battery be of strength ′𝑉′ volts. Let the
combined resistance of the two resistors be ′𝑅′ ohms.

Therefore, according to Ohm's law, we have


𝑉 = 𝐼𝑅 –––––––––––––––– (1)
Let the potential difference across ′𝑅1 ′ is ′𝑉1 ′ and across ′𝑅2 ′ is ′𝑉2 ′.
𝑉1 = 𝐼𝑅1 ––––––––––––––– (2)
𝑉2 = 𝐼𝑅2 ––––––––––––––– (3)

We know that when the resistors are connected in series, the current is the same in all the
resistors but voltage is not. Therefore
𝑉 = 𝑉1 + 𝑉2 ––––––––– (4)
Putting the equations (1), (2) and (3) in equation (4) we get–
𝐼𝑅 = 𝐼𝑅1 + 𝐼𝑅2
𝑅 = 𝑅1 + 𝑅2
132) Let the current be ′𝐼′ amperes and the battery be of strength ′𝑉′ volts. Let the combined
resistance of the three resistors be ′𝑅′ ohms.
Therefore, according to Ohm's law, we have
𝑉 = 𝐼𝑅 ––––––––––––––––– (1)
Let the potential difference across ′𝑅1 ′ is ′𝑉1 ′ , across ′𝑅2 ′ is ′𝑉2 ′ and across ′𝑅3 ′ is ′𝑉3 ′.
𝑉1 = 𝐼𝑅1 –––––––––––––––– (2)
𝑉2 = 𝐼𝑅2 –––––––––––––––– (3)
𝑉3 = 𝐼𝑅3 –––––––––––––––– (4)

We know that when the resistors are connected in series, the current is the same in all the
resistors but voltage is not. Therefore,
𝑉 = 𝑉1 + 𝑉2 + 𝑉3 ––––––––– (5)

Putting equations (1), (2), (3) and (4) in equation (5), we get–
𝐼𝑅 = 𝐼𝑅1 + 𝐼𝑅2 + 𝐼𝑅3
⇒ 𝑅 = 𝑅1 + 𝑅2 + 𝑅3
133) Suppose total current flowing in the circuit is ′𝐼′ then the current passing through
resistance ′𝑅1 ′ will be ′𝐼1 ′ and current passing through resistance ′𝑅2 ′ will be ′𝐼2 ′.

Total current: 𝐼 = 𝐼1 + 𝐼2 ––––––––––––– (1)


Let resultant resistance of this parallel combination is ′𝑅′. By applying the Ohm's law to the
whole circuit, we get that
𝑉
𝐼 = 𝑅 –––––––––––––––– (2)

Since the potential difference across both the resistances is same, so applying Ohm's law to
each resistance, we get–
𝑉
𝐼1 = 𝑅 –––––––––––––– (3)
1

𝑉
𝐼2 = –––––––––––––– (4)
𝑅2

Putting equations (2), (3) and (4) in the equation (1), we get–
𝑉 𝑉 𝑉
= +
𝑅 𝑅1 𝑅2
1 1 1
= +
𝑅 𝑅1 𝑅2
∴ If two resistance are connected in parallel, then the resultant resistance will be–
1 1 1
= +
𝑅 𝑅1 𝑅2
134) Suppose total current flowing in the circuit is ′𝐼′ , then the current passing through
resistance ′𝑅1 ′ will be ′𝐼1 ′ current passing through resistance ′𝑅2 ′ will be ′𝐼2 ′ and
current passing through resistance ′𝑅3 ′ will be ′𝐼3 ′.

Total current: 𝐼 = 𝐼1 + 𝐼2 + 𝐼3 ––––––––––––– (1)


Let resultant resistance of this parallel combination is ′𝑅′ . By applying Ohm's law to the
whole circuit, we get–
𝑉
𝐼= –––––––––––––––– (2)
𝑅

Since the potential difference across all the resistances is same, so applying Ohm's law to
each resistance, we get–
𝑉
𝐼1 = –––––––––––––– (3)
𝑅1
𝑉
𝐼2 = 𝑅 –––––––––––––– (4)
2

𝑉
𝐼3 = 𝑅 –––––––––––––– (5)
3

Putting equations (2), (3), (4) and (5) in equation (1), we get –
𝑉 𝑉 𝑉 𝑉
= + +
𝑅 𝑅1 𝑅2 𝑅3
1 1 1 1
= + +
𝑅 𝑅1 𝑅2 𝑅3
∴ If three resistances are connected in parallel, then the resultant resistance will be–
1 1 1 1
= + +
𝑅 𝑅1 𝑅2 𝑅3
135) Let the experimental setup comprises of three resistors 𝑅1 , 𝑅2 and 𝑅3 of three different
values such as 1 Ω, 2 Ω and 3 Ω which are connected in series. Connect them with a battery
of 6 𝑉, an ammeter and plug a key as shown in the figure.

The key ′𝐾′ is closed and the ammeter reading is recorded. Now, the position of ammeter
is changed to anywhere in between the resistors again and again and the ammeter reading
is recorded each time. It is observed that there was identical reading each time, which
shows that same current flows through every part of the circuit containing three
resistances in series connected to a battery.
136) The experimental setup comprises of three resistors 𝑅1 , 𝑅2 and 𝑅3 which are joined in
parallel combinations and connecting them with a battery, an ammeter, a voltmeter and a
plug key ′𝐾′ as shown in the figure.
The key ′𝐾′ is closed and the voltmeter and ammeter readings are recorded.
The key ′𝐾′ is open and removing the ammeter and voltmeter from the circuit and insert
the voltmeter in parallel with ′𝑅1 ′ and ammeter in series with resistor ′𝑅1 ′ as shown in
figure. Again, the voltmeter and ammeter readings are recorded.

Similarly, measuring the potential differences across resistances ′𝑅2 ′ and ′𝑅3 ′. It is found
that voltmeter gives identical readings which leads to conclude that the voltage or
potential difference across each resistors is same and equal to the potential difference
across the combination.
137) 1 ohm
138) 1 ohm
139) In case of parallel combination, the resultant resistance will be less than either of the
individual resistances.
140) The equivalent resistance will be less than 1 Ω. This is because when a number of
resistances are connected in parallel, then their equivalent resistance is less than the
smallest individual resistance.
141)
a) For minimum current, these two should be connected in series.
b) For maximum current, these two should be connected in parallel.
142) 8 Ω if connected in series and 1.5 Ω if connected in parallel.
143) Since the resultant resistance is less than the individual resistances, so the resistances
should be connected in parallel.
144)
a) In parallel b) In series
145)
a) Connect 2 Ω resistor in series with a parallel combination of 3 Ω and 6 Ω resistors.
b) Connect 2 Ω, 3 Ω and 6 Ω resistors in parallel.
146)
a) Connect a 6 Ω resistor in series with a parallel combination of two 6 Ω resistors.
b) Connect a parallel combination of three 6 Ω resistors with another parallel
combination of three 6 Ω resistors.
147) Connect the 5 Ω resistor in parallel with a series combination of 2 Ω and 3 Ω resistors.
148)
a) Connect all three resistors in series.
b) Connect all three resistors in parallel.
c) Connect the 3 Ω resistor in parallel with a series combination of 1 Ω and 2 Ω resistors.
149) Combine two 100 Ω resistors in series with a parallel combination of two 100 Ω resistors.
150)
a) 48 Ω b) 2Ω
151) Smallest = 0.01 Ω and Largest = 100 Ω
152) 12 Ω
153) 3 Ω and 6 Ω
154) 4 resistors
𝑅
155)
4

156) 25 : 1
157)
a) 1500 Ω b) 1Ω c) 5Ω
158) 9.17 A when connected separately, 4.58 A when connected in series and 18.34 A when
connected in parallel.
159) 0.67 A
160)
a) Resistors are connected in parallel.
b) Potential difference across each resistor is same and is equal to 6 𝑉.
c) 2 ohms resistance have bigger share of current because of its lower resistance.
d) 1.2 Ω e) 5𝐴
161) Current across 𝑅1 is 4 𝐴 and current across 𝑅2 is 6 𝐴.
162) The circuit diagram containing a cell, a key, a resistor of 2 Ω in series with a combination of
two resistors 4 Ω each in parallel and a voltmeter across the parallel combination is shown
below.

The resistance of parallel combination of two resistors (4 Ω each) is 2 Ω. The same current
passes through the 2 Ω resistor and 2 Ω (equivalent of two parallel resistors of 4 Ω ). So,
voltage drop by them will be same, hence the voltage V is same across 2 Ω resistor and the
parallel combination of 2 Ω resistors, cell of 4 Ω.
163) 𝑅𝐿 = 5 Ω. Current through lamp and conductor of 5 Ω in series remains the same. i.e. 1 A.
Also, Potential difference across lamp remains the same. i.e. 5 V.
164) 31.25 Ω and 7.04 A
4
165) Ω And current through 2 Ω coil is 2A
3

166)

a) 24 Ω b) 0.25 A c) 5V d) 1V
167)
a)

b) 40 Ω c) 0.125 A d) 2.5 V
168)
a) 2Ω b) 3A c) 2A
169)
a) 8Ω b) 0.5 A c) 3V
170)
a) 9.16 A b) 4.58 A c) 18.33 A
171)
a) 0.6 A and 0.4 A b) 5 V and 11 V c) 11 Ω
172)
a) 0.44 Ω b) 3.2 V
173)
a) 6Ω b) 2A c) 8V
174)
a) Current through 𝑅1 is 2.4 A, Current through 𝑅2 is 1.2 A, Current through 𝑅3 is 0.4 A
b) 4A c) 3Ω
175)
a) 18 Ω b) 0.67 A
176)
a) 5Ω b) 2A c) 4V d) 6V
177)
a) 2Ω b) 12 V c) 12 V d) 4A e) 2A
178) 1.25 A and 0.5 A
179)
a) 2A b) 2A c) 3Ω d) 16 V
180)
a) Current through 𝑅1 is 1.2 A, Current through 𝑅2 is 0.6 A, Current through 𝑅3 is 0.2 A
b) 2A c) 3 Ω
181)
a) 2.5 Ω b) 1.6 A
182) 16 Ω
183) 0.9 A
184) The total current of 0.5 A flowing in the circuit distributes equally in the two arms having
lamps (since the lamps have same resistances). So, the current through each of these arms
is 0.25 A. Hence 𝐴2 , 𝐴3 , 𝐴4 𝑎𝑛𝑑 𝐴5 , all will read 0.25 A.
185) The advantages of connecting electrical devices in parallel or disadvantages of connecting
electrical devices in series are the following –
a) In parallel circuits, if one electrical appliance stops working due to some defect, then
all other appliances keep working normally. On the other hand, in series circuit, if one
electrical appliance stops working due to some defect, then all other appliances also
stop working because the whole circuit is broken.
b) In parallel circuits, each electrical appliance has its own switch due to which it can be
turned on or off independently, without affecting other appliances. In series circuit,
all the electrical appliances have only one switch due to which they cannot be turned
on or off independently.
c) In parallel circuits, each electrical appliance gets the same voltage as that of the
battery due to which all the appliances work properly. In series circuit, the appliances
do not get the same voltage as that of the battery because the voltage is shared by all
the appliances.
d) In the parallel connection of electrical appliances, the overall resistance of the circuit
is reduced due to which the current from the battery is high and hence each
electrical appliance can draw the required amount of current. In the series
connection of electrical appliances, the overall resistance of the circuit increases too
much due to which the current from the battery is low and hence all the electrical
appliances cannot draw sufficient current for their proper working.
186) A series arrangement is not used for connecting domestic electrical appliances in a circuit
because of the following disadvantages –
a) In series circuit, if one electrical appliance stops working due to some defect, then all
other appliances also stop working because the whole circuit is broken.
b) In series circuit, all the electrical appliances have only one switch due to which they
cannot be turned on or off independently.
c) In series circuit, the appliances do not get the same voltage as that of the battery
because the voltage is shared by all the appliances.
d) In the series connection of electrical appliances, the overall resistance of the circuit
increases too much due to which the current from the battery is low and hence all
the electrical appliances cannot draw sufficient current for their proper working.
187) The advantages of connecting electrical devices in parallel are the following –
a) In parallel circuits, if one electrical appliance stops working due to some defect, then
all other appliances keep working normally.
b) In parallel circuits, each electrical appliance has its own switch due to which it can be
turned on or off independently, without affecting other appliances.
c) In parallel circuits, each electrical appliance gets the same voltage as that of the
battery due to which all the appliances work properly.
d) In the parallel connection of electrical appliances, the overall resistance of the circuit
is reduced due to which the current from the battery is high and hence each
electrical appliance can draw the required amount of current.
188) All the other bulbs also stop glowing.
189) All the other bulbs keep glowing.
190) All the other lamps stop glowing.
191) All lamps are connected in series.
192) No, they are wired in parallel.
193) Parallel arrangement, because if one electrical bulb stops glowing due to some defect, the
other will keep glowing.
194)
a) Series b) Parallel
195) Parallel arrangement takes more current from the battery due to its lesser equivalent
resistance.
196) The brightness of two lamps arranged in parallel is much more than those arranged in
series.
197)
a) Circuit (ii) b) Circuit (iii) c) Circuit (iii)
198)
a) Parallel circuit b) Parallel circuit
c) Series circuit d) Series circuit
199)

200)

201)
a) C will be the brightest. Voltage will be distributed equally between A and B, so they
will have equal brightness but lesser than that of C.
b) A gets the same voltage as before, so its brightness remains the same.
c) If B burns out, A will also stop glowing because it is connected in series with B.
However, brightness of C remains the same.
202)
a) Turn the switch to right side so as the resistance decreases.
b) Turn the switch to left side so as the resistance increases.
203) The rate at which electrical work is done or the rate at which electrical energy is consumed,
is called electric power. Formulae:
𝑊
𝑃=
𝑡

where, 𝑊 = work done

𝑡 = time taken

𝑃 = 𝑉𝐼
where, 𝑉 = potential difference
𝐼 = current
𝑃 = 𝐼2𝑅
where, 𝐼 = current
𝑅 = resistance
𝑉2
𝑃=
𝑅
where, 𝑉 = potential difference
𝑅 = resistance
204) When an electrical appliance consumes electrical energy at the rate of 1 joule per second,
its power is said to be 1 watt. 1 Watt = 1 Volt × 1 Ampere.
205) Power.
206) Watt.
207) 60 watt bulb, because power is inversely proportional to the resistance.
208) If the potential difference between the ends of a wire of fixed resistance is doubled, then
the power will become four times as power is directly proportional to the square of
potential difference.
209) Electrical energy consumed by an electrical appliance is given by the product of its power
rating and the time for which it is used.
210) Electrical energy consumed by an electrical appliance depends on:
a) Power rating of the appliance.
b) Time for which the appliance is used.
211) By reducing the length of the element, the resistance will decrease and because power is
inversely proportional to resistance, therefore, this will result in more consumption of
energy.
212) Joule.
213) Kilowatt-hour.
214)
a) Electric power. b) Electric energy.
215) kWh is the short form of kilowatt-hour. It is the commercial unit of electrical energy.
216) When an electrical appliance of 1 watt power is used for 1 hour, then the amount of
electrical energy consumed is said to be 1 watt-hour. 1 watt-hour = 3600 joules.
217) When an electrical appliance of 1000 watt (1 kilowatt) power is used for 1 hour, then the
amount of electrical energy consumed is said to be 1 kilowatt-hour. 1 kilowatt-hour =
3600000 joules (or 3.6 × 106 Joules).

218) In case of series circuit,

Let us assume that the resistance of each bulb is ′𝑅′ and potential difference is ′𝑉′.

Equivalent resistance in series combination= 𝑅𝑒𝑞 = 𝑅 + 𝑅 + 𝑅 = 3𝑅

Let current through each bulb in series combination be 𝐼1.

𝑉
Now, 𝑉 = 𝐼1 × 3𝑅 ⇒ 𝐼1 = 3𝑅

Power consumption across each bulb in series combination is,


𝑉 2 𝑉2
𝑃1 = 𝐼12 (3𝑅) = ( ) × 3𝑅 = × 3𝑅
3𝑅 9𝑅 2
𝑉2
⇒ 𝑃1 = 3𝑅 ___________________ (1)

In case of parallel circuit,


Resistance of each bulb = 𝑅 and
Voltage across each bulb = 𝑉
∴ Power consumption of each bulb in parallel combination is,
𝑉2
𝑃2 = __________________ (2)
𝑅
𝑃 (𝑉 2 ⁄3𝑅)
From equations (1) and (2), we have, 𝑃1 = (𝑉 2 ⁄𝑅 )
⇒ 𝑃2 = 3𝑃1
2

So, brightness of each bulb in parallel combination will increase. Each bulb will glow 3 times
brighter to that of each bulb in series combination.
𝑉2
219) As we know that, Electric power consumed , 𝑃 =
𝑅
Here ′𝑉′ is given, we have to find the value of ′𝑅′.
When the connection is in parallel, the equivalent resistance is:
1 1 1 2 𝑅
= + = ⇒ 𝑅𝑒𝑞 =
𝑅𝑒𝑞 𝑅 𝑅 𝑅 2
220 × 220 96800
Electric power consumed in parallel connection = =
𝑅 ⁄2 𝑅

When the connected is in series, the equivalent resistance is:


𝑅𝑒𝑞 = 𝑅 + 𝑅 = 2𝑅
220 × 220 24200
Electric power consumed in series connection = =
𝑅 𝑅

𝑃𝑝𝑎𝑟𝑎𝑙𝑙𝑒𝑙 96800⁄𝑅 4
Now, the ratio of electric power consumed by them = = =
𝑃𝑠𝑒𝑟𝑖𝑒𝑠 24200⁄𝑅 1

220) Other information is that it will consume energy at the rate of 36 J/s.
221) 100 W
222) 110 W
223) 4 A
224) 2.99 kW
225) 484 Ω
226)
a) 6Ω b) 1.5 W
227) 18 × 106 J
228) (𝑏) A current of 2 A passed through a resistance of 100 ohms will have more electrical
energy.
229) TV set uses 0.25 kWh energy whereas toaster uses 0.20 kWh energy. So, TV uses more
energy.
230) 1.1 kW and 2.2 kWh
231) 6.6 kWh and 10 A
232)
a) 2300 W or 2.3 kW b) 138000 J or 138 kJ
233)
a) 18.18 A b) 12.1 Ω c) 8 kWh d) ₹ 36.80
234) 59.4 kWh
235)
a)

b) 0.45 A c) 356.4 kJ
236) ₹ 39
237)
a) 42919 b) 42935 c) 16 units
d) 24 hours e) ₹ 80
238) ₹ 55
239) 92 J/s
240)
a) Lamp, because least current is flowing in it.
b) Large current drawn by kettle and earth connection needed.
c) 2040 d) 4.25 A
241) 110 bulbs
242)
a) When bulb 𝐵1 gets fused, the currents in 𝐵2 and 𝐵3 remain same (i.e. 1 A), because
voltage across 𝐵2 and 𝐵3 bulb remains the same, so their glow remains unaltered.
b) When bulb 𝐵2 gets fused, the current in 𝐵2 becomes zero and current
in 𝐵1 and 𝐵3 remains 1 A, because voltage across 𝐵1 and 𝐵3 bulb remains the same.
c) 13.5 W
243) 3 Ampere in the resistor A and 1.5 Ampere in the resistors B and C.
244)
a) 8W b) 8W
245)
1
a) Ω b) 1A c) 4V d) 4W
4

e) There is no difference in the readings of ammeters 𝐴1 and 𝐴2 as same current flows


through all elements in a series current.
246) When an electric current is passed through a high resistance wire, like nichrome wire, the
resistance wire becomes very hot and produces heat. This is called the heating effect of
current. The heating effect of current is obtained by the transformation of electrical energy
into heat energy. E.g. electric iron, electric heater, etc.
247) Assuming that all electrical work done or electrical energy consumed is converted into heat
energy. i.e. heat produced. So, heat produced is given by 𝐻 = 𝐼 2 𝑅𝑡 where heat is produced
in Joule’s when a current of ′𝐼′ Amperes flows in a wire of resistance ′𝑅′ ohms for
time ′𝑡′ seconds.
Thus, it is known as Joule’s law of heating.
This law implies that heat produced in a resistor is directly proportional to:
a) Square of current.
b) Resistance of wire.
c) Time for which the current flows through the resistor.

In a conductor, when an electric field is applied across its ends, the free electrons available
in it start drifting opposite to the direction of the electric field. These electrons collide with
the atoms which have lost the electrons. As a result of these collisions, some energy of
electrons is transferred to the atoms which vibrate violently as they gain energy. Thus, heat
is developed in the conductor. Greater the current, greater will be the rate of collision and
so greater will be the heat produced.

248) 𝐻 = 𝐼 2 𝑅𝑡.
249) All materials offer resistance to the flow of current through them. So some external energy
is required to make the current flow. This energy is provided by the battery. Some of this
energy gets dissipated as heat energy, so the resistor becomes hot.
Work done in carrying a charge ‘Q’ through a potential difference ‘V’ is given as
𝑊 = 𝑉 × 𝑄 –––––––––––– (1)
Also, we know that 𝑄 = 𝐼 × 𝑡 –––––––––––– (2)
Using Ohm's law, 𝑉 = 𝐼 × 𝑅 –––––––––––––– (3)
Putting (2) and (3) in (1), we get–
𝑊 = 𝐼 2 𝑅𝑡
This work done in carrying the charge through the wire appears as the heat produced.
i.e. 𝐻 = 𝐼 2 𝑅𝑡
This energy is dissipated as heat energy.
This law is called Joule’s law of heating effect.
250) 𝐻 = 𝐼 2 𝑅𝑡. i.e. Heat produced by an electric current is directly proportional to the following
factors:
a) Square of current: Since heat is directly proportional to the square of current, so if
we double the current, then the heat produced will become four times and if we
halve the current, then the heat produced will become one-fourth.
b) Resistance of wire: Since heat produced in a wire is directly proportional to the
resistance, so if we double the resistance, then heat produced will also become
double and if we halve the resistance, then heat produced will also become halved.
c) Time for which current is passed: Since heat produced in a wire is directly
proportional to the time for which current flows, so if we double the time period for
which the current is passed, then heat produced will also become double and if we
halve the time period, then heat produced will also become halved.
251) Heat produced is directly proportional to the square of current. So, if we double the
current, then the heat produced will become four times and if we halve the current, then
the heat produced will become one-fourth.
252) If current is doubled, heat will become four times because heat produced is directly
proportional to the square of current.
253) If current is halved, heat will become one-fourth because heat produced is directly
proportional to the square of current.
254) Since heat produced in a wire is directly proportional to the resistance and total resistance
in series connection is more than that in parallel connection, so heat obtained per minute is
more when they are connected in series as compared to when they are connected in
parallel.
255) Three important applications of the heating effect of electric current are given below:
a) The heating effect of electric current is utilised in the working of electrical heating
appliances such as electric iron, electric kettle, electric toaster, electric oven, room
heaters, water heaters (geysers), etc. All these heating appliances contain coils of
high resistance wire made of nichrome alloy. When these appliances are connected
to power supply by insulated copper wires then a large amount of heat is produced in
the heating coils (because they have high resistance), but a negligible heat is
produced in the connecting wires of copper (because copper has very, very low
resistance). E.g. The heating element (or coil) of an electric iron made of nichrome
glows because it becomes red hot due to the large amount of heat produced on
passing current (because of its high resistance), but the cord or connecting cable of
the electric iron made of copper does not glow because negligible heat is produced in
it by passing current (because of its extremely lower resistance). The temperature of
the heating element (or heating coil) of an electric heating device when it becomes
red hot and glows is about 900° C.
b) The heating effect of current is utilised in electric bulbs (electric lamps) for producing
light. When electric current passes through a very thin, high resistance tungsten
filament of an electric bulb, the filament becomes white hot and emits light. The
same current is flowing through the tungsten filament of an electric bulb produces
enormous heat but almost negligible heat is produced in the connecting wires of
copper. This is because of the fact that the fine tungsten filament has very high
resistance whereas copper connecting wires have very low resistance.

Tungsten metal is used for making the filaments of electric bulbs because it has a
very high melting point (of 3380° C). Due to its very high melting point, the tungsten
filament can be kept white hot without melting away. The other properties of
tungsten which making it suitable for making electric bulbs are its high flexibility and
low rate of evaporation at high temperatures.

When the tungsten filament of electric bulb becomes white hot and glows to emit
light, then its temperature is about 2500° C. If air is present in an electric bulb, then
the extremely hot tungsten filament would burn up quickly in the oxygen of air. So,
the electric bulb is filled with chemically unreactive gas like argon or nitrogen or a
mixture of both.
c) The heating effect of electric current is utilised in electric fuse for protecting
household wiring and electrical appliances. A fuse is a short length of a thin tin plated
copper wire having low melting point. The thin fuse wire has a higher resistance than
the rest of the electric wiring in a house. So, when the current in a household electric
circuit rises too much due to some reason, then the fuse wire gets heated too much,
melts and breaks the circuit (due to which the current stops flowing). This prevents
the fire in house (due to overheating of wiring) and also prevents damage to various
electrical appliances in the house due to excessive current flowing through them.
256) Tungsten metal is used for making the filaments of electric bulbs because it has a very high
melting point (of 3380° C). Due to its very high melting point, the tungsten filament can be
kept white hot without melting away. The other properties of tungsten which making it
suitable for making electric bulbs are its high flexibility and low rate of evaporation at high
temperatures.
257) When the tungsten filament of electric bulb becomes white hot and glows to emit light,
then its temperature is about 2500° C. If air is present in an electric bulb, then the
extremely hot tungsten filament would burn up quickly in the oxygen of air. So, the electric
bulb is filled with chemically unreactive gas like argon or nitrogen or a mixture of both.
258) The connecting cord of the heater is made of a metal (such as copper) and it does not glow
because negligible heat is produced in it by passing current (because copper has extremely
low resistance and heat produced is directly proportional to resistance) but the heating
element is made of an alloy (such as nichrome) and it glows because it becomes red-hot
due to the large amount of heat produced on passing current (because nichrome has high
resistance and heat produced is directly proportional to resistance).
259) The connecting wires of the heater get only slightly warm because they have extremely low
resistance and heat produced is directly proportional to the resistance due to which
negligible heat is produced in them by passing current.
260)
a) S, because it has high resistivity. b) Q, because it has very low resistivity.
c) R, because it has very high resistivity.
261) Tungsten.
262) Tungsten is used almost exclusively for making the filaments of electric lamps (or electric
bulbs) because it has very high melting point (of 3380° C) due to which it can be kept white
hot without melting away. Also, tungsten has high flexibility and low rate of evaporation at
high temperature.
263) The filament wire becomes white hot where as other wires in the circuit do not get heated
much. High resistance of filament wire accounts for this difference because heat produced
is directly proportional to resistance.
264) Argon and Nitrogen.
265) The electric bulb is filled with a chemically unreactive gas like argon or nitrogen because
these gases do not react with the hot tungsten filament and hence prolong the life of the
filament of the bulb. If air is filled in an electric bulb, then the extremely hot tungsten
filament would burn up quickly in the oxygen of air.
266) Filament type electric bulbs are not power efficient because most of the electric power
consumed by the filament of a bulb appears as heat and only a small amount of electric
power is converted in light.
267) Heating effect and magnetic effect are the two effects which are produced by electric
current.
268) Heating effect.
269) Heating effect.
270) Heating effect.
271) Electric heater, filament type electric bulb and electric fuse.
272) Electric iron, electric oven, water heater, room heater, electric fuse, etc.
273) 15000 J
274) 14520 J
275) 20 V
276) 4.8× 106 Joules
277)
a) 48 W b) 12 V
278) 1250 J/s
279) 1800 J
280) 345.6 J
281) 720 J
282) 1.64 A , 134.15 Ω ; 3.82 A , 57.60 Ω
283) 𝑎 284) 𝑑 285) 𝑏
286) 𝑏 287) 𝑎 288) 𝑐
289) 𝑑 290) 𝑐 291) 𝑑
292) 𝑏 293) 𝑑 294) 𝑐
295) 𝑎 296) 𝑑 297) 𝑑
298) 𝑏 299) 𝑑 300) 𝑐
301) 𝑐 302) 𝑑 303) 𝑐
304) 𝑏 305) 𝑑 306) 𝑑
307) 𝑐 308) 𝑑 309) 𝑐
310) 𝑐 311) 𝑏 312) 𝑏
313) 𝑑 314) 𝑐 315) 𝑐
316) 𝑑 317) 𝑐 318) 𝑑
319) 𝑏 320) 𝑏 321) 𝑑
322) 𝑏 323) 𝑎 324) 𝑏
325) 𝑑 326) 𝑎 327) 𝑐
328) 𝑐 329) 𝑏 330) 𝑏
331) 𝑐 332) 𝑏 333) 𝑎
334) 𝑑 335) 𝑏 336) 𝑏
337) 𝑎 338) 𝑐 339) 𝑑
340) 𝑑 341) 𝑑 342) 𝑐
343) 𝑑 344) 𝑐 345) 𝑐
346) 𝑏 347) 𝑐 348) 𝑑
349) 𝑏 350) 𝑐 351) 𝑐
352) 𝑐 353) 𝑐 354) 𝑑
355) 𝑐 356) 𝑑 357) 𝑐
358) 𝑑 359) 𝑎 360) 𝑏
361) 𝑐 362) 𝑐 363) 𝑎
364) 𝑐 365) 𝑎 366) 𝑎
367) 𝑏 368) 𝑏 369) 𝑏
370) True 371) True 372) False
373) False 374) True
375) volts, voltmeter, parallel 376) conductor, insulator
377) electrons, closed 378) amperes, ammeter, series
379) current 380) ohms, increases, increases, decreases

You might also like